IASbaba’s Daily CSAT Practice Test
ARCHIVES
Daily CSAT Practice Test
Everyday 5 Questions from Aptitude, Logical Reasoning, and Reading Comprehension will be covered from Monday to Saturday.
Make the best use of the initiative. All the best!
To Know More about Ace the Prelims (ATP) 2021 – CLICK HERE
Important Note:
- Don’t forget to post your marks in the comment section. Also, let us know if you enjoyed today’s test 🙂
- After completing the 5 questions, click on ‘View Questions’ to check your score, time taken and solutions.
Test-summary
0 of 5 questions completed
Questions:
- 1
- 2
- 3
- 4
- 5
Information
To view Solutions, follow these instructions:
- Click on – ‘Start Test’ button
- Solve Questions
- Click on ‘Test Summary’ button
- Click on ‘Finish Test’ button
- Now click on ‘View Questions’ button – here you will see solutions and links.
You have already completed the test before. Hence you can not start it again.
Test is loading...
You must sign in or sign up to start the test.
You have to finish following test, to start this test:
Results
0 of 5 questions answered correctly
Your time:
Time has elapsed
You have scored 0 points out of 0 points, (0)
Average score |
|
Your score |
|
Categories
- Not categorized 0%
Pos. | Name | Entered on | Points | Result |
---|---|---|---|---|
Table is loading | ||||
No data available | ||||
- 1
- 2
- 3
- 4
- 5
- Answered
- Review
-
Question 1 of 5
1. Question
Sandeep starts a business with Rs. 36,000. After a certain period of time he is joined by Dhanu, who invests Rs. 27,000. At the end of the year they divide the profit in the ratio of 8: 3. For what period did Dhanu join Sandeep?
Correct
Solution (c)
Ratio of profit = capital of Sandeep*time/capital of dhanu*time
Let Deepak’s investment be for x months.
Then, 8/3 = (36,000*12) / (27,000*X)
On solving X = 6 months
Incorrect
Solution (c)
Ratio of profit = capital of Sandeep*time/capital of dhanu*time
Let Deepak’s investment be for x months.
Then, 8/3 = (36,000*12) / (27,000*X)
On solving X = 6 months
-
Question 2 of 5
2. Question
In a class of 35 students, Ziva is placed 7th from the bottom whereas Sindhu is placed 9th from the top. Sharan is placed in between the two. What is Ziva’s position from Sharan?
Correct
Solution (a)
As seen in the figure, Sharan is between Sindhu and Ziva.
It’s given that Ziva is 7th from the bottom and Sindhu is 9th from the top.
Therefore, number of persons between Sindhu and Ziva = 35 – (9 + 7) = 19
Sharan’s position between Sindhu and Ziva = (19+1)/2 = 10
Hence, Sharan is at the middle i.e. at 10th position from both. Ziva, therefore, is at the 10th position from Sharan
Incorrect
Solution (a)
As seen in the figure, Sharan is between Sindhu and Ziva.
It’s given that Ziva is 7th from the bottom and Sindhu is 9th from the top.
Therefore, number of persons between Sindhu and Ziva = 35 – (9 + 7) = 19
Sharan’s position between Sindhu and Ziva = (19+1)/2 = 10
Hence, Sharan is at the middle i.e. at 10th position from both. Ziva, therefore, is at the 10th position from Sharan
-
Question 3 of 5
3. Question
In a certain language, if SUNSHINE is coded as TVOTIJOF then how will MOON be coded:
Correct
Solution (b)
Increase every letter by 1.
Incorrect
Solution (b)
Increase every letter by 1.
-
Question 4 of 5
4. Question
Consider the given statement and the two conclusions that follow. You have to answer which of them are implicit in the statement.
Statement: It is desirable to put the child in school at the age of 5 or so.
Conclusions:
- At that age the child reaches appropriate level of development and is ready to learn.
- The schools do not admit children after six years of age.
Choose the correct option.
Correct
Solution (a)
Since the statement talks of putting the child in school at the age of 5, it means that the child is mentally prepared for the same at this age. So, I is implicit. But nothing about admission after 6 years of age is mentioned in the statement. So, II is not implicit.
Incorrect
Solution (a)
Since the statement talks of putting the child in school at the age of 5, it means that the child is mentally prepared for the same at this age. So, I is implicit. But nothing about admission after 6 years of age is mentioned in the statement. So, II is not implicit.
-
Question 5 of 5
5. Question
Consider the given statement and the two conclusions that follow. You have to answer which of them are implicit in the statement.
Statement: Unemployment allowance should be given to all unemployed Indian youth above 18 years of age.
Conclusions:
- There are unemployed youth in India who needs monetary support
- The government has sufficient funds to provide allowance to all unemployed youth
Choose the correct option.
Correct
Solution (a)
I directly follows from the statement and so is implicit. Also, the statement is a suggestion and does not tell about a government policy or its position of funds. So, II is not implicit
Incorrect
Solution (a)
I directly follows from the statement and so is implicit. Also, the statement is a suggestion and does not tell about a government policy or its position of funds. So, II is not implicit